Pedi Flashcards

1
Q

A 16-hour-old male neonate was presented to a
hospital with a complaint of failure to suck of five
hours duration. He was delivered in a hospital at
gestational age of 35-week. On examination, he has
grunting, respirations were 68/min, pulse rate was
178/min, and has pale skin color. On laboratory
investigations, hemoglobin level was 5g/dl, and
blood group is A+ and 0+ for him and his mother
respectively. What is the most likely cause of the
neonate’s condition?
• a. Rh incompatibility
• b. ABO incompatibility
• c. Vitamin K deficiency
• d. Anemia of prematurity

A

The most likely cause of the neonate’s condition is:

b. ABO incompatibility

Explanation:
• Clinical Features: The neonate has pale skin, grunting, tachypnea (respirations 68/min), tachycardia (pulse 178/min), and severe anemia (hemoglobin 5 g/dL). These symptoms are consistent with hemolysis and anemia.
• Maternal and Neonatal Blood Groups: The mother’s blood group is O+, and the neonate’s blood group is A+. This is a classic setup for ABO incompatibility, where the mother’s anti-A or anti-B antibodies cross the placenta and attack the neonate’s red blood cells, causing hemolysis and anemia.
• Rh Incompatibility (Option a): Rh incompatibility is less likely because both mother and neonate are Rh-positive, and this condition typically occurs in an Rh-negative mother with an Rh-positive baby.
• Vitamin K Deficiency (Option c): While vitamin K deficiency can cause bleeding, it does not explain the severe anemia or the hemolytic findings.
• Anemia of Prematurity (Option d): While prematurity can cause anemia due to insufficient erythropoiesis, the rapid onset (16 hours post-birth) and severity of anemia (hemoglobin 5 g/dL) point more strongly to hemolysis from ABO incompatibility.

Summary:

The findings of hemolysis, severe anemia, and the blood group mismatch (O+ mother and A+ neonate) strongly support ABO incompatibility as the most likely cause.

How well did you know this?
1
Not at all
2
3
4
5
Perfectly
2
Q

A 10-minute-old baby was delivered from a 30-year-
old P-III mother with an assisted vacuum delivery.The baby’s fifth minuteAPGAR score was 5 and is
currently on resuscitation with bag and face mask
ventilation. The heart rate is 55/min. What is the
most appropriate next step in the management of
the newborn?
a. Continue to resuscitate with bag and face
mask
• b. Stop resuscitation and give the baby to
parents
• c. Intubation and Epinephrine
• d. Chest compressions coordinated with PPV

A

The correct answer is: d. Chest compressions coordinated with PPV (Positive Pressure Ventilation).

Explanation:

In neonatal resuscitation, if the heart rate remains <60/min after 30 seconds of effective positive pressure ventilation (PPV) with a bag and mask, the next step is to initiate chest compressions coordinated with PPV. This follows the Neonatal Resuscitation Program (NRP) guidelines:
1. Initial Steps: Ensure adequate ventilation with proper technique (effective mask seal and appropriate ventilation pressures).
2. Assess Heart Rate:
• If ≥100/min: Continue supportive care.
• If 60-99/min: Continue effective ventilation.
• If <60/min despite 30 seconds of effective PPV: Proceed to chest compressions.
3. Chest Compressions: Compressions should be coordinated with PPV at a ratio of 3:1 (3 compressions to 1 ventilation) at a rate of 120 events per minute (90 compressions + 30 breaths).
4. Reassess Heart Rate after 60 seconds of chest compressions and ventilation:
• If still <60/min, administer epinephrine and consider intubation.

Why not the other options?
• a. Continue to resuscitate with bag and face mask: If the heart rate is still <60/min, continuing PPV alone is inadequate. Chest compressions are necessary.
• b. Stop resuscitation and give the baby to parents: This is not appropriate unless the resuscitation is deemed futile (e.g., no signs of life after prolonged efforts), which is not the case here.
• c. Intubation and Epinephrine: These are not the immediate next steps. Epinephrine is indicated only if the heart rate remains <60/min despite 60 seconds of coordinated chest compressions and PPV.

Key Takeaway:

For a newborn with a heart rate <60/min after effective ventilation, initiate chest compressions coordinated with PPV as the next step.

How well did you know this?
1
Not at all
2
3
4
5
Perfectly
3
Q

A 30-minute-old male neonate was presented with a
complaint of fast breathing for 10 minutes. On
physical examination, the neonate is in respiratory
distress, respiration rate is 75/min, pulse rate is
154/min, temperature is 36.5 degree centigrade and
gestational age is 43 weeks. Chest X-ray shows
patchy infiltrates. What is the most likely diagnosis?
• a. Neonatal sepsis
• b. Birth asphyxia
• c. Meconium aspiration syndrome
• d. Hyaline membrane disease

A

C -because baby is born at 43 weeks , the clinical presentation plus the X-ray findings go with it

How well did you know this?
1
Not at all
2
3
4
5
Perfectly
4
Q

A female neonate, who was delivered at gestational
age of 43 weeks, has been admitted toa NICU for the
last five days after being referred due to respiratory
distress immediately after delivery. Physical
examination findings at admission revealed over-
distended chest and Meconium stained extremities.
Currently, she has severe hypoxia with clear chest
and good air entry. Otherwise, she has no detected
abnormality. The chest radiograph is normal. What is
the most likely diagnosis for the current problem?
a. Spontaneous tension Pneumothorax
• b. Bacterial superimposed infection
• c. Congenital diaphragmatic hernia
• d. Persistent pulmonary hypertension

A

The correct answer is: d. Persistent Pulmonary Hypertension (PPHN).

Explanation:

Clinical Features Supporting PPHN:
1. Post-term delivery (43 weeks): PPHN is more common in post-term infants due to factors like chronic uteroplacental insufficiency, meconium aspiration syndrome, or birth asphyxia.
2. Respiratory distress shortly after birth: This is a hallmark of PPHN due to failure of the pulmonary vasculature to transition from high resistance (fetal circulation) to low resistance (neonatal circulation).
3. Severe hypoxia with normal chest examination and radiograph: PPHN causes hypoxemia due to right-to-left shunting through the ductus arteriosus and/or foramen ovale, despite clear lungs and good air entry.
4. Meconium-stained extremities: Suggests a history of meconium aspiration syndrome, a common trigger for PPHN.

Why Not the Other Options?
• a. Spontaneous tension pneumothorax:
• A pneumothorax would present with asymmetrical chest findings, diminished air entry on the affected side, and signs of tension (e.g., tracheal deviation). None of these are present here, and the chest radiograph is normal.
• b. Bacterial superimposed infection:
• A bacterial infection would likely present with systemic signs (e.g., fever, lethargy) and abnormal radiographic findings (e.g., infiltrates or consolidation). This neonate has a clear chest and normal radiograph.
• c. Congenital diaphragmatic hernia:
• This condition typically presents with severe respiratory distress immediately after birth, scaphoid abdomen, and abnormal radiographic findings (e.g., bowel loops in the thoracic cavity). None of these features are present here.

Pathophysiology of PPHN:
• In PPHN, pulmonary vascular resistance remains high, causing right-to-left shunting through the ductus arteriosus and/or foramen ovale, leading to hypoxemia despite adequate ventilation and a structurally normal heart and lungs.

Management:
• Oxygenation: Optimize oxygen delivery with high-flow oxygen or mechanical ventilation.
• Pulmonary vasodilators: Medications like inhaled nitric oxide (iNO) or sildenafil may reduce pulmonary vascular resistance.
• Supportive care: Maintain normal temperature, glucose levels, and acid-base balance.

Key Point:

The combination of post-term gestation, meconium exposure, severe hypoxia with normal radiograph and chest exam, and no other significant abnormalities strongly suggests persistent pulmonary hypertension of the newborn (PPHN).

How well did you know this?
1
Not at all
2
3
4
5
Perfectly
5
Q

P Flag question
A female neonate, who was delivered at gestational
age of 43 weeks, has been admitted toa NICU for the
last five days after being referred due to respiratory
distress immediately after delivery. Physical
examination findings at admission revealed over-
distended chest and Meconium stained extremities.
Currently, she has severe hypoxia with clear chest
and good air entry. Otherwise, she has no detected
abnormality. The chest radiograph is normal. What is
the most likely diagnosis for the current problem?
a. Spontaneous tension Pneumothorax
• b. Bacterial superimposed infection
• c. Congenital diaphragmatic hernia
• d. Persistent pulmonary hypertension
Previous page

A

The correct answer is: d. Persistent Pulmonary Hypertension (PPHN).

Explanation:

Clinical Features Supporting PPHN:
1. Post-term delivery (43 weeks): PPHN is more common in post-term infants due to factors like chronic uteroplacental insufficiency, meconium aspiration syndrome, or birth asphyxia.
2. Respiratory distress shortly after birth: This is a hallmark of PPHN due to failure of the pulmonary vasculature to transition from high resistance (fetal circulation) to low resistance (neonatal circulation).
3. Severe hypoxia with normal chest examination and radiograph: PPHN causes hypoxemia due to right-to-left shunting through the ductus arteriosus and/or foramen ovale, despite clear lungs and good air entry.
4. Meconium-stained extremities: Suggests a history of meconium aspiration syndrome, a common trigger for PPHN.

Why Not the Other Options?
• a. Spontaneous tension pneumothorax:
• A pneumothorax would present with asymmetrical chest findings, diminished air entry on the affected side, and signs of tension (e.g., tracheal deviation). None of these are present here, and the chest radiograph is normal.
• b. Bacterial superimposed infection:
• A bacterial infection would likely present with systemic signs (e.g., fever, lethargy) and abnormal radiographic findings (e.g., infiltrates or consolidation). This neonate has a clear chest and normal radiograph.
• c. Congenital diaphragmatic hernia:
• This condition typically presents with severe respiratory distress immediately after birth, scaphoid abdomen, and abnormal radiographic findings (e.g., bowel loops in the thoracic cavity). None of these features are present here.

Pathophysiology of PPHN:
• In PPHN, pulmonary vascular resistance remains high, causing right-to-left shunting through the ductus arteriosus and/or foramen ovale, leading to hypoxemia despite adequate ventilation and a structurally normal heart and lungs.

Management:
• Oxygenation: Optimize oxygen delivery with high-flow oxygen or mechanical ventilation.
• Pulmonary vasodilators: Medications like inhaled nitric oxide (iNO) or sildenafil may reduce pulmonary vascular resistance.
• Supportive care: Maintain normal temperature, glucose levels, and acid-base balance.

Key Point:

The combination of post-term gestation, meconium exposure, severe hypoxia with normal radiograph and chest exam, and no other significant abnormalities strongly suggests persistent pulmonary hypertension of the newborn (PPHN).

How well did you know this?
1
Not at all
2
3
4
5
Perfectly
6
Q

P Flag question
A five-year-old female child with complaints of fever
and rash of three days duration is brought to a
hospital. She also has cough, nasal discharge and
diarrhea. On physical examination, her temperature
is 37.8 degree centigrade, respirations are 43/min,
pulse rate is 123/min, and there is diffuse red
maculo-papular rash from face to leg. What
complication will be expected in her third decade of
life?
• a. Pneumonia
• b. Severe malnutrition
• c. Subacute sclerosing panencephalitis
• d. Measles encephalitis

A

The correct answer is: d. Persistent Pulmonary Hypertension (PPHN).

Explanation:

Clinical Features Supporting PPHN:
1. Post-term delivery (43 weeks): PPHN is more common in post-term infants due to factors like chronic uteroplacental insufficiency, meconium aspiration syndrome, or birth asphyxia.
2. Respiratory distress shortly after birth: This is a hallmark of PPHN due to failure of the pulmonary vasculature to transition from high resistance (fetal circulation) to low resistance (neonatal circulation).
3. Severe hypoxia with normal chest examination and radiograph: PPHN causes hypoxemia due to right-to-left shunting through the ductus arteriosus and/or foramen ovale, despite clear lungs and good air entry.
4. Meconium-stained extremities: Suggests a history of meconium aspiration syndrome, a common trigger for PPHN.

Why Not the Other Options?
• a. Spontaneous tension pneumothorax:
• A pneumothorax would present with asymmetrical chest findings, diminished air entry on the affected side, and signs of tension (e.g., tracheal deviation). None of these are present here, and the chest radiograph is normal.
• b. Bacterial superimposed infection:
• A bacterial infection would likely present with systemic signs (e.g., fever, lethargy) and abnormal radiographic findings (e.g., infiltrates or consolidation). This neonate has a clear chest and normal radiograph.
• c. Congenital diaphragmatic hernia:
• This condition typically presents with severe respiratory distress immediately after birth, scaphoid abdomen, and abnormal radiographic findings (e.g., bowel loops in the thoracic cavity). None of these features are present here.

Pathophysiology of PPHN:
• In PPHN, pulmonary vascular resistance remains high, causing right-to-left shunting through the ductus arteriosus and/or foramen ovale, leading to hypoxemia despite adequate ventilation and a structurally normal heart and lungs.

Management:
• Oxygenation: Optimize oxygen delivery with high-flow oxygen or mechanical ventilation.
• Pulmonary vasodilators: Medications like inhaled nitric oxide (iNO) or sildenafil may reduce pulmonary vascular resistance.
• Supportive care: Maintain normal temperature, glucose levels, and acid-base balance.

Key Point:

The combination of post-term gestation, meconium exposure, severe hypoxia with normal radiograph and chest exam, and no other significant abnormalities strongly suggests persistent pulmonary hypertension of the newborn (PPHN).

How well did you know this?
1
Not at all
2
3
4
5
Perfectly
7
Q

Flag question
A four-year-old male child was brought by his mother
to a clinic accompanied with a complaint of being
unable to speak clear words. He clearly points to the
thing he wants to have. He is able to ride tricycle and
copies cross and circle on paper. Routine physical
examinations did not show any abnormality. What is
the most appropriate measure to be taken for the
baby?
• a. Referring the child for mental retardation
a s s e s s m e n t
O b. Reassuring the mother to send him to
regular school
c. Reassuring the mother as it is normal
• d. Referring the child for hearing assessment

A

The correct answer is: d. Referring the child for hearing assessment.

Explanation:

The child’s inability to speak clear words despite otherwise normal development (e.g., pointing, riding a tricycle, and copying shapes) raises concern about a potential speech or language delay. One of the most common and reversible causes of delayed speech development is hearing impairment. Even mild or intermittent hearing loss (e.g., due to recurrent otitis media) can significantly affect language acquisition.

Why refer for hearing assessment?
1. Speech Delay:
• The inability to speak clear words at 4 years old is concerning, as most children at this age can form clear sentences and express themselves verbally.
2. Normal Physical Examination:
• With no other abnormalities, the first step is to rule out hearing impairment, which is a key factor in language development.
3. Normal Motor Skills and Cognition:
• The child’s ability to ride a tricycle and copy a cross and circle indicates age-appropriate fine and gross motor skills, making hearing assessment the most logical next step.

Why not the other options?
• a. Referring for mental retardation assessment:
• The child’s cognitive abilities seem intact based on their ability to perform age-appropriate tasks like copying shapes and riding a tricycle. There is no evidence of intellectual disability here.
• b. Reassuring the mother to send him to regular school:
• While the child might attend school, the speech issue needs to be addressed first to ensure effective communication and learning.
• c. Reassuring the mother as it is normal:
• Delayed speech at 4 years old is not normal and requires investigation.

Key Point:

The most appropriate step is to refer the child for hearing assessment, as early identification and intervention for hearing loss are critical to improving language development and overall communication skills.

How well did you know this?
1
Not at all
2
3
4
5
Perfectly
8
Q

A mother has brought her three-month-old male
infant to a clinic to get advice about his growth
status. Anthropometry is done and all parameters
are unaffected. What is the normal growth pattern to
inform the woman?
• a. Head circumference increases by 2 cm per
month for the next three months
• b. Weight increases by 30 g/day for the next
t h r e e m o n t h s
c. Length increases by 0.5 cm per month for
the next three months
• d. Weight doubles the birth weight at 4-5
months

A

The correct answer is: d. Weight doubles the birth weight at 4-5 months.

Explanation:

At three months of age, the infant’s growth typically follows these patterns:
1. Weight:
• Infants gain approximately 20–30 grams/day during the first 3–4 months.
• By 4–5 months, the infant’s weight usually doubles the birth weight.
2. Length:
• The length increases by 2.5 cm/month during the first 6 months, not just 0.5 cm/month (making option c incorrect).
3. Head Circumference:
• The head circumference increases by about 1 cm/month for the first 6 months, not 2 cm/month after three months (making option a incorrect).
4. Growth milestones:
• Weight gain slows down after 6 months, and by 12 months, the weight typically triples the birth weight.
• Length increases more slowly after 6 months, reaching about 50% of birth length by 1 year.

Why not the other options?
• a. Head circumference increases by 2 cm per month for the next three months:
• This rate of growth is too rapid after the neonatal period. It is about 1 cm/month from 3–6 months.
• b. Weight increases by 30 g/day for the next three months:
• While 30 g/day is appropriate for the first 3 months, the rate begins to slow slightly after this period.
• c. Length increases by 0.5 cm per month for the next three months:
• The length increases at about 2.5 cm/month during the first 6 months, much faster than stated here.

Key Counseling Point for the Mother:

The infant’s weight is expected to double the birth weight by 4–5 months and triple by 12 months. Both weight and height should continue to be monitored at regular check-ups to ensure normal growth and development.

How well did you know this?
1
Not at all
2
3
4
5
Perfectly
9
Q

A nine-hour-old male neonate was brought to a
hospital with a complaint of failure to suck since
birth. He was delivered at home and did not cry
immediately after delivery. On physical examination,
he has seizure on arrival; all neonatal reflexes are
depressed, and lethargic. What is the first line
treatment for this neonate?
a. Phenytoin
b. Phenobarbital
c. Valproic acid
• d. Diazepam

A

The correct answer is: b. Phenobarbital.

Explanation:

This neonate’s presentation of seizures, lethargy, and depressed neonatal reflexes suggests neonatal encephalopathy, likely due to birth asphyxia (hypoxic-ischemic encephalopathy, HIE) given the history of failure to cry immediately after birth.

First-line treatment for neonatal seizures:
• Phenobarbital is the first-line drug for managing seizures in neonates because:
• It is effective in controlling neonatal seizures.
• It has a well-established safety profile in neonates.
• It is the preferred anticonvulsant according to neonatal resuscitation and seizure management guidelines.

Why not the other options?
• a. Phenytoin:
• Phenytoin is considered a second-line treatment for neonatal seizures if phenobarbital fails.
• c. Valproic acid:
• Valproic acid is contraindicated in neonates because of the risk of hepatotoxicity and other adverse effects.
• d. Diazepam:
• Diazepam is not commonly used in neonates for seizure management because it has a short duration of action and can cause respiratory depression, especially in this age group.

Additional Steps:
• Supportive care: Ensure adequate oxygenation, ventilation, and glucose levels (correct hypoglycemia if present).
• Investigation: Identify and treat the underlying cause of seizures (e.g., hypoxia, hypoglycemia, infection, metabolic disturbances).

Key Point:

Phenobarbital is the first-line treatment for neonatal seizures due to its efficacy, safety, and long history of use in neonates.

How well did you know this?
1
Not at all
2
3
4
5
Perfectly
10
Q

A nine-hour-old male neonate was brought to a
hospital with a complaint of failure to suck since
birth. He was delivered at home and did not cry
immediately after delivery. On physical examination,
he has seizure on arrival; all neonatal reflexes are
depressed, and lethargic. What is the first line
treatment for this neonate?
a. Phenytoin
b. Phenobarbital
c. Valproic acid
• d. Diazepam

A

The correct answer is: b. Phenobarbital.

Explanation:

This neonate’s presentation of seizures, lethargy, and depressed neonatal reflexes suggests neonatal encephalopathy, likely due to birth asphyxia (hypoxic-ischemic encephalopathy, HIE) given the history of failure to cry immediately after birth.

First-line treatment for neonatal seizures:
• Phenobarbital is the first-line drug for managing seizures in neonates because:
• It is effective in controlling neonatal seizures.
• It has a well-established safety profile in neonates.
• It is the preferred anticonvulsant according to neonatal resuscitation and seizure management guidelines.

Why not the other options?
• a. Phenytoin:
• Phenytoin is considered a second-line treatment for neonatal seizures if phenobarbital fails.
• c. Valproic acid:
• Valproic acid is contraindicated in neonates because of the risk of hepatotoxicity and other adverse effects.
• d. Diazepam:
• Diazepam is not commonly used in neonates for seizure management because it has a short duration of action and can cause respiratory depression, especially in this age group.

Additional Steps:
• Supportive care: Ensure adequate oxygenation, ventilation, and glucose levels (correct hypoglycemia if present).
• Investigation: Identify and treat the underlying cause of seizures (e.g., hypoxia, hypoglycemia, infection, metabolic disturbances).

Key Point:

Phenobarbital is the first-line treatment for neonatal seizures due to its efficacy, safety, and long history of use in neonates.

How well did you know this?
1
Not at all
2
3
4
5
Perfectly
11
Q

A seven-month-old infant is brought to a hospital for
a routine follow up. Growth and development of the
infant are optimal. A WBC count of 7,000/mm3,
hemoglobin level of 6.7g/dl with MCV of 60fl, and
RDW of 20percent were noted from laboratory work
up. What is the most likely diagnosis of the child?
• a. Thalassemia
• b. Lead poisoning
• c. Iron-deficiency anemia
• d. Folate deficiency anemia

A

The correct answer is: c. Iron-deficiency anemia.

Explanation:

This 7-month-old infant presents with laboratory findings suggestive of microcytic anemia, most likely due to iron-deficiency anemia (IDA), which is the most common cause of anemia in infants and young children.

Supporting Features for Iron-Deficiency Anemia:
1. Low hemoglobin (6.7 g/dL):
• Significantly low for age (normal: ≥10.5 g/dL at 6–12 months).
2. Low mean corpuscular volume (MCV = 60 fL):
• Indicates microcytic anemia (normal MCV: 70–86 fL for this age).
3. High red cell distribution width (RDW = 20%):
• Elevated RDW indicates variability in red blood cell (RBC) size, which is typical in IDA.
4. Optimal growth and development:
• Suggests nutritional causes (like iron deficiency) rather than genetic disorders like thalassemia, which might affect growth.
5. Age (7 months):
• This is a common age for iron deficiency to manifest, especially in infants transitioning to complementary feeding, as breast milk alone may no longer meet iron requirements.

Why Not the Other Options?
• a. Thalassemia:
• While thalassemia also causes microcytic anemia, RDW is usually normal in thalassemia, as RBC size is uniformly small. Additionally, thalassemia is less likely in a healthy child without a family history or physical abnormalities.
• b. Lead poisoning:
• Lead poisoning can cause microcytic anemia, but it is less common and is usually associated with environmental exposure and basophilic stippling on a peripheral blood smear, which is not mentioned here.
• d. Folate deficiency anemia:
• Folate deficiency causes macrocytic anemia, with an increased MCV, not microcytic anemia.

Key Point:

The combination of microcytic anemia, elevated RDW, and age strongly suggests iron-deficiency anemia, which should prompt dietary evaluation and possible iron supplementation.

How well did you know this?
1
Not at all
2
3
4
5
Perfectly
12
Q

A seven-year-old known cardiac patient, with
atrioventricualr septal defect and who is on follow
up, is planning to undergo dental correction with
manipulation of gingival tissue. He has no known
history of drug allergy. What is the best first-line
endocarditis preventive strategy for this child?
• a. Cefepime
b. Amoxicillin
c. Azithromycin
d. Clindamycin

A

The correct answer is: b. Amoxicillin.

Explanation:

Children with congenital heart defects, such as an atrioventricular septal defect (AVSD), are at increased risk for infective endocarditis during certain dental procedures that involve manipulation of gingival tissue. This is because bacteria from the mouth can enter the bloodstream and infect the heart valves or the endocardial lining. Therefore, prophylactic antibiotics are recommended before dental procedures.

Best Choice for Endocarditis Prophylaxis:
• Amoxicillin is the first-line antibiotic for endocarditis prophylaxis in children without any known drug allergies. It is effective against the most common organisms that cause infective endocarditis (such as viridans group streptococci), and it is the preferred drug for dental procedures in patients at risk for endocarditis.

Dosing for Amoxicillin:
• The usual dose for children is 50 mg/kg given orally, typically 30–60 minutes before the procedure.

Why Not the Other Options?
• a. Cefepime:
• Cefepime is a broad-spectrum cephalosporin typically used for more severe infections, but it is not the first choice for routine endocarditis prophylaxis in this case. Amoxicillin is preferred due to its proven efficacy against the organisms typically responsible for infective endocarditis in dental procedures.
• c. Azithromycin:
• Azithromycin can be used in patients who are allergic to penicillin, but it is not the first-line agent. It is generally used when the patient has a documented penicillin allergy.
• d. Clindamycin:
• Clindamycin is another alternative for patients allergic to penicillin, but again, it is not the first choice for endocarditis prophylaxis. Like azithromycin, it is typically reserved for those with penicillin allergy.

Key Point:

For a child with no known drug allergies who is undergoing a dental procedure with manipulation of gingival tissue, the best first-line prophylactic antibiotic is amoxicillin.

How well did you know this?
1
Not at all
2
3
4
5
Perfectly
13
Q

A seven-year-old known cardiac patient, with
atrioventricualr septal defect and who is on follow
up, is planning to undergo dental correction with
manipulation of gingival tissue. He has no known
history of drug allergy. What is the best first-line
endocarditis preventive strategy for this child?
• a. Cefepime
b. Amoxicillin
c. Azithromycin
d. Clindamycin

A

The correct answer is: b. Amoxicillin.

Explanation:

Children with congenital heart defects, such as an atrioventricular septal defect (AVSD), are at increased risk for infective endocarditis during certain dental procedures that involve manipulation of gingival tissue. This is because bacteria from the mouth can enter the bloodstream and infect the heart valves or the endocardial lining. Therefore, prophylactic antibiotics are recommended before dental procedures.

Best Choice for Endocarditis Prophylaxis:
• Amoxicillin is the first-line antibiotic for endocarditis prophylaxis in children without any known drug allergies. It is effective against the most common organisms that cause infective endocarditis (such as viridans group streptococci), and it is the preferred drug for dental procedures in patients at risk for endocarditis.

Dosing for Amoxicillin:
• The usual dose for children is 50 mg/kg given orally, typically 30–60 minutes before the procedure.

Why Not the Other Options?
• a. Cefepime:
• Cefepime is a broad-spectrum cephalosporin typically used for more severe infections, but it is not the first choice for routine endocarditis prophylaxis in this case. Amoxicillin is preferred due to its proven efficacy against the organisms typically responsible for infective endocarditis in dental procedures.
• c. Azithromycin:
• Azithromycin can be used in patients who are allergic to penicillin, but it is not the first-line agent. It is generally used when the patient has a documented penicillin allergy.
• d. Clindamycin:
• Clindamycin is another alternative for patients allergic to penicillin, but again, it is not the first choice for endocarditis prophylaxis. Like azithromycin, it is typically reserved for those with penicillin allergy.

Key Point:

For a child with no known drug allergies who is undergoing a dental procedure with manipulation of gingival tissue, the best first-line prophylactic antibiotic is amoxicillin.

How well did you know this?
1
Not at all
2
3
4
5
Perfectly
14
Q

A six-week-old male infant was brought to a hospital
with a complaint of rash for one hour. He took his
first vaccine 12 hours back. On physical
examination, his pulse rate is feeble, respirations are
65/min, he has stridor and wheezing over the chest,
diffuse urticurial rash all over the body, and lip and
oral mucosa swelling. What is the appropriate initial
treatment for this infant?
• a. Blood transfusion
• b. Prednisolone
c. Antihistamine
d. Epinephrine

A

The correct answer is: d. Epinephrine.

Explanation:

The infant’s presentation of feeble pulse, tachypnea (respirations 65/min), stridor, wheezing, diffuse urticarial rash, and swelling of lips and oral mucosa is highly suggestive of anaphylaxis, a severe and potentially life-threatening allergic reaction. This likely occurred as a reaction to the vaccine administered 12 hours earlier.

First-line treatment for anaphylaxis:
• Epinephrine is the drug of choice for anaphylaxis. It acts rapidly to:
• Relieve airway obstruction by reducing bronchospasm, laryngeal edema, and mucosal swelling.
• Improve cardiovascular status by increasing heart rate, cardiac output, and vasoconstriction to counteract hypotension.

Dose of Epinephrine:
• Intramuscular (IM) injection:
• Dose: 0.01 mg/kg (maximum 0.3 mg per dose) of 1:1000 (1 mg/mL) solution.
• Administered into the mid-anterolateral thigh.
• Repeat every 5–15 minutes as needed, based on response.

Why Not the Other Options?
• a. Blood transfusion:
• There is no indication for a blood transfusion in this scenario. The symptoms are due to anaphylaxis, not blood loss or anemia.
• b. Prednisolone:
• Corticosteroids, like prednisolone, can be used as an adjunct treatment to prevent biphasic reactions, but they have a delayed onset of action (hours). They are not appropriate as initial treatment for anaphylaxis.
• c. Antihistamine:
• Antihistamines (e.g., diphenhydramine) can help with skin symptoms (e.g., rash, itching) but do not address the life-threatening respiratory or cardiovascular components of anaphylaxis. They are secondary treatments, not initial ones.

Key Point:

Epinephrine IM is the initial and life-saving treatment for anaphylaxis. Other supportive measures, like oxygen, fluid resuscitation, and adjunctive medications (e.g., antihistamines and corticosteroids), may be used after stabilization.

How well did you know this?
1
Not at all
2
3
4
5
Perfectly
15
Q

A three-year-old boy presented with inadequate
intake and diarrhea of three days duration. His
growth and development were optimal. On
examination, skin pinch goes back very slowly and
he is lethargic. His weight is 15kg. What is the most
appropriate fluid therapy to this child?
• a. 1500ml RL over 24 hours
• b. 450ml RL over 30 minutes, followed by
1050ml over the next 2½ hours
• c. 450ml RL over 1 hour, followed by 1050ml
over next 5 hours
• d. 300ml RL rapid bolus, followed by 450ml RL
over 30minutes

A

The answer is B . The child is 3 years old so hydrate with 100 ml /kg … 30 ml/kg the first 30 minutes and 70 ml/kg the next 2.5 hrs
If the child is am Infant it is given over 1 hr then 5 hrs

How well did you know this?
1
Not at all
2
3
4
5
Perfectly
16
Q

P Flag question
A three-year-old female child was presented to a
hospital with a complaint of fear of light
(photophobia) for three months. She also has skin
rash more on the arm, leg and buttocks areas. On
physical examination, she has foamy appearing
plaques on conjunctiva and dry, scaly, hyperkeratotic
patches over the leg, arm and buttocks. What is the
most likely diagnosis?
• a. Thiamine deficiency
• b. Vitamin A deficiency
• c. Zinc deficiency
• d. Riboflavin deficiency

A

The correct answer is: b. Vitamin A deficiency.

Explanation:

The child’s presentation is characteristic of Vitamin A deficiency, which is commonly associated with:
1. Photophobia (fear of light): Due to impaired retinal function and early eye involvement in vitamin A deficiency.
2. Foamy plaques on the conjunctiva (Bitot’s spots): A hallmark sign of vitamin A deficiency.
3. Dry, scaly, hyperkeratotic skin (follicular hyperkeratosis): Caused by abnormal keratinization of epithelial tissues, commonly on the arms, legs, and buttocks.

Why Not the Other Options?
• a. Thiamine deficiency:
• Thiamine deficiency causes beriberi, which is characterized by neurological symptoms (dry beriberi) or cardiovascular issues (wet beriberi). It does not cause eye or skin findings as described here.
• c. Zinc deficiency:
• Zinc deficiency can cause acrodermatitis enteropathica, presenting with periorificial and acral dermatitis, diarrhea, and alopecia, but it does not cause Bitot’s spots or photophobia.
• d. Riboflavin deficiency:
• Riboflavin deficiency causes symptoms such as angular stomatitis, glossitis, and corneal vascularization, but it does not lead to conjunctival plaques or follicular hyperkeratosis.

Key Points:
• Vitamin A deficiency primarily affects the eyes (night blindness, photophobia, and Bitot’s spots) and skin (follicular hyperkeratosis).
• It is most common in children with malnutrition or poor dietary intake, especially in developing countries.
• Treatment involves vitamin A supplementation and improvement of the child’s diet to include foods rich in vitamin A, such as liver, eggs, dairy, and dark green or orange vegetables.

How well did you know this?
1
Not at all
2
3
4
5
Perfectly
17
Q

A three-year-old male child was presented to a
hospital with a complaint of fever for five days. He
also has a yellowish discoloration of the eye and
frequent abnormal body movement of one day
duration. He is from malaria endemic area. On
physical examination, icteric sclera, splenomegaly
and hepatomegly were found. What is the best first
line treatment for this child?
• a. Artesunate
b. Coartum
• c. Quinine
• d. Arthemeter

A

The correct answer is: a. Artesunate.

Explanation:

This child has symptoms consistent with severe malaria, including:
1. Fever for five days: A hallmark of malaria.
2. Yellowish discoloration of the eye: Indicates jaundice, often due to hemolysis or liver dysfunction caused by severe malaria.
3. Frequent abnormal body movements: Suggests seizures, likely due to cerebral malaria.
4. Splenomegaly and hepatomegaly: Common findings in severe malaria due to increased immune response and sequestration of infected red blood cells in the spleen.

Given these features, the child meets the criteria for severe malaria, which requires parenteral therapy with a rapidly acting antimalarial drug.

First-Line Treatment:
• Parenteral Artesunate is the drug of choice for severe malaria in children, as per WHO guidelines. It is more effective and has fewer side effects compared to quinine.
• Dosage: Administer intravenously or intramuscularly, usually at 2.4 mg/kg at 0, 12, and 24 hours, followed by once daily until the patient can tolerate oral therapy.

Why Not the Other Options?
• b. Coartem (artemether-lumefantrine):
• Coartem is an oral therapy used for uncomplicated malaria. It is not suitable for severe malaria or when the patient cannot tolerate oral medications.
• c. Quinine:
• Quinine is an alternative for severe malaria but is less effective than artesunate. It has a higher risk of side effects, including hypoglycemia and cardiotoxicity, especially in children.
• d. Artemether:
• Artemether is an intramuscular drug but is less preferred compared to artesunate for severe malaria because artesunate has superior efficacy in reducing mortality.

Key Points:
• For severe malaria, the first-line treatment is parenteral artesunate.
• After the patient improves and can tolerate oral medications, the treatment is switched to a complete course of oral antimalarial therapy (e.g., Coartem).
• Supportive care, including seizure control and management of complications like hypoglycemia, jaundice, or dehydration, is essential.

How well did you know this?
1
Not at all
2
3
4
5
Perfectly
18
Q

A three-year-old male child was presented to a
hospital witha complaint of generalized body
swelling of five days duration. On examination, he
had puffy face, and his blood pressurewas between
the 50th and 90th percentile. On urinalysis, +3protein
was revealed. Cholesterol level of 400 mg/di,
triglyceride level of 250 mg/d|,serum creatinine level
of 0.4mg/dl, and BUN level of 15 mg/d were noted
on laboratory investigations. What is the most
appropriate management for the child?
• a. Start Amoxcillin 40 mg/kg/day
• b. Start Furosemide 4 mg/kg/day
• c. Start Enalapril 200 microgram/kg/day
• d. Start Prednisolone 60 mg/m2/day

A

The correct answer is: d. Start Prednisolone 60 mg/m²/day.

Explanation:

This child presents with clinical and laboratory findings consistent with nephrotic syndrome, characterized by:
1. Generalized body swelling (edema): Puffy face is a classic finding.
2. Proteinuria (+3 on urinalysis): A hallmark of nephrotic syndrome.
3. Hypercholesterolemia (400 mg/dL) and hypertriglyceridemia (250 mg/dL): Common in nephrotic syndrome due to increased hepatic lipid synthesis.
4. Normal blood pressure: Suggests minimal kidney injury.
5. Normal serum creatinine (0.4 mg/dL) and BUN (15 mg/dL): Indicates preserved kidney function.

Management:

The first-line treatment for idiopathic nephrotic syndrome (most commonly Minimal Change Disease) in children is high-dose corticosteroids, specifically:
• Prednisolone at a dose of 60 mg/m²/day, given daily for 4-6 weeks, followed by a gradual taper over several weeks to avoid relapse.

Corticosteroids help reduce proteinuria by modulating the immune response, which is thought to contribute to the permeability changes in the glomerular basement membrane.

Why Not the Other Options?
• a. Start Amoxicillin 40 mg/kg/day:
• There is no indication for antibiotics unless there is evidence of an infection, such as peritonitis or cellulitis, which are complications of nephrotic syndrome.
• b. Start Furosemide 4 mg/kg/day:
• Diuretics like furosemide may be used to manage severe edema, but they are adjunctive and not first-line therapy. They should be used cautiously to prevent intravascular volume depletion.
• c. Start Enalapril 200 microgram/kg/day:
• ACE inhibitors like enalapril are used in hypertensive patients with proteinuria. This child has normal blood pressure, so it is not indicated here.

Key Points:
• Idiopathic nephrotic syndrome in children is most commonly due to Minimal Change Disease.
• The cornerstone of treatment is corticosteroids.
• Monitor for complications (e.g., infections, thromboembolism) and side effects of steroids during treatment.

How well did you know this?
1
Not at all
2
3
4
5
Perfectly
19
Q

A two-hour-old male neonate was brought to a NICU
after delivered from Hepatitis B surface antigen
positive mother. On physical examination, his weight
is 1500 gm; otherwise no other pertinent finding was
detected. What is the most appropriate vaccination
time of hepatitis B vaccine for this neonate?
• a. After weight reach 2.5kg
b. c. At six weeks of age
Within 12 hour of birth
• d. After one month of age

A

The correct answer is: c. Within 12 hours of birth.

Explanation:

Neonates born to mothers who are Hepatitis B surface antigen (HBsAg)-positive are at high risk of acquiring perinatal hepatitis B infection, which can lead to chronic liver disease. Immediate postnatal prophylaxis is critical, regardless of the neonate’s birth weight.

Recommended Management:
1. Within 12 hours of birth:
• Administer Hepatitis B vaccine (first dose).
• Administer Hepatitis B immunoglobulin (HBIG) at a different injection site to provide passive immunity.
2. Follow-up doses of the hepatitis B vaccine:
• A total of three doses should be administered as per the routine immunization schedule (e.g., at 0, 1–2 months, and 6 months of age).
3. Testing for hepatitis B status:
• Perform HBsAg and anti-HBs testing at 9–12 months to confirm the success of immunization and absence of infection.

Why Not the Other Options?
• a. After weight reaches 2.5 kg:
• Delaying vaccination until the neonate gains weight increases the risk of hepatitis B transmission.
• b. At six weeks of age:
• Waiting until six weeks significantly delays protection and puts the neonate at risk of early infection.
• d. After one month of age:
• Delaying the first dose beyond 12 hours of life is not recommended as it compromises the effectiveness of preventing perinatal transmission.

Key Points:
• For neonates born to HBsAg-positive mothers, Hepatitis B vaccine and HBIG must be administered within 12 hours of birth, regardless of birth weight or gestational age.
• The timing is crucial to prevent perinatal hepatitis B transmission.

How well did you know this?
1
Not at all
2
3
4
5
Perfectly
20
Q

A two-month-old male infant presented with fast
breathing and feeding interruption of two weeks
duration. On examination, his pulse rate is 160/min,
respirations are 60/min, temperature is 37.3 degree
centigrade, oxygen saturation is 95percent, he has
sub-costal and inter-costal retractions and total liver
span is 9cm. What is the most likely diagnosis of
this child?
a. Pneumonia
b. Perinatal tuberculosis
C. Large VSD
d. Severe a n e m i a

A

The correct answer is: c. Large VSD (Ventricular Septal Defect).

Explanation:

This infant presents with signs of heart failure, likely due to a large ventricular septal defect (VSD):
1. Fast breathing and feeding interruption:
• Symptoms of congestive heart failure in infants.
• Feeding interruption occurs because of fatigue from increased respiratory effort.
2. Respiratory rate of 60/min with sub-costal and intercostal retractions:
• Suggestive of respiratory distress due to pulmonary congestion caused by increased blood flow to the lungs (left-to-right shunting through the VSD).
3. Liver span of 9 cm:
• Hepatomegaly is a common finding in heart failure in children.
4. Pulse rate of 160/min:
• A compensatory tachycardia due to increased cardiac workload.
5. Oxygen saturation of 95%:
• Normal oxygen saturation indicates no significant hypoxemia, which makes cyanotic heart diseases less likely.

Why Not the Other Options?
• a. Pneumonia:
• While respiratory distress is present, pneumonia would typically be associated with fever, crepitations, or decreased air entry on chest examination. These are not mentioned in this case.
• b. Perinatal tuberculosis:
• Tuberculosis in neonates typically presents with systemic symptoms such as fever, weight loss, lymphadenopathy, and possibly hepatosplenomegaly, but respiratory distress alone is not characteristic.
• d. Severe anemia:
• Severe anemia can cause tachycardia and fatigue but does not usually cause hepatomegaly or respiratory distress unless it leads to heart failure, which would still be secondary to another underlying condition.

Key Points:
• Large VSD leads to left-to-right shunting, causing pulmonary overcirculation, respiratory distress, and signs of heart failure in infants.
• Management includes:
• Medical therapy: Diuretics and afterload reduction to manage heart failure.
• Nutritional support: To meet increased metabolic demands.
• Definitive surgical repair: Usually planned when the child is older, but may be expedited if symptoms are severe.

How well did you know this?
1
Not at all
2
3
4
5
Perfectly
21
Q

A two-year-old female child presented to a hospital
with a complaint offensive nasal discharge of three
days duration. She also has a history of low-grade
fever, cough and bad breath. Physical examination
revealed bilateral erythematous and swollen nasal
mucosa with purulent discharge. What is the most
likely diagnosis in this child?
O a. Sinusitis
• b. Foreign body
c. Lymphadenitis
• d. Periodontitis

A

The correct answer is: a. Sinusitis.

Explanation:

This child’s presentation is consistent with acute sinusitis, which is common in young children due to the anatomy and function of their upper respiratory tract.

Key Features:
1. Offensive nasal discharge of three days duration:
• Purulent nasal discharge is a hallmark of sinusitis.
2. Low-grade fever and cough:
• Fever and cough often accompany sinus infections due to inflammation and mucus drainage into the respiratory tract.
3. Bad breath (halitosis):
• Caused by bacterial overgrowth in stagnant mucus.
4. Physical examination findings:
• Bilateral erythematous and swollen nasal mucosa with purulent discharge strongly supports sinusitis.

Why Not the Other Options?
• b. Foreign body:
• Nasal foreign body often presents with unilateral purulent and foul-smelling nasal discharge, not bilateral symptoms.
• c. Lymphadenitis:
• Lymphadenitis typically presents with swollen lymph nodes, localized tenderness, and possibly systemic symptoms. There’s no mention of lymph node involvement here.
• d. Periodontitis:
• Periodontitis involves gum inflammation and oral symptoms (e.g., gingival swelling, bleeding, or pain). It doesn’t cause nasal discharge or swollen nasal mucosa.

Key Points:
• Acute sinusitis is a common complication of viral upper respiratory tract infections in children.
• Diagnosis is based on clinical symptoms, including purulent nasal discharge, nasal congestion, and facial pain or pressure.
• Management includes supportive care, and antibiotics may be indicated if symptoms persist for more than 10 days, worsen after initial improvement, or are severe from the outset.

How well did you know this?
1
Not at all
2
3
4
5
Perfectly
22
Q

A two-hour-old male neonate was brought to a NICU
because he had fast breathing. He was delivered at
38 weeks via cesarean delivery with birth weight of
3500 gm and Apgar score of 8 and 10 in the first and
fifth minutes respectively. Examination revealed a
respiratory rate of 65/min in otherwise vigorous
baby. What is the most likely diagnosis for this
baby?
a. Congenital pneumonia
• b. Pulmonary hypoplasia
O c. Transient tachypnea of the newborn
• d. Meconium aspiration syndrome

A

The correct answer is: c. Transient tachypnea of the newborn (TTN).

Explanation:

This neonate presents with tachypnea (respiratory rate of 65/min), which is common in the immediate postnatal period. However, the baby is otherwise vigorous and well-appearing, which points towards a diagnosis of Transient Tachypnea of the Newborn (TTN).

Key Features:
1. Fast breathing (tachypnea) in a term neonate (38 weeks):
• TTN typically occurs in term or near-term infants, especially those delivered by cesarean section.
2. Birth weight of 3500 gm and Apgar scores of 8 and 10:
• These suggest a well-grown and otherwise healthy baby, ruling out more severe conditions like congenital pneumonia or meconium aspiration syndrome.
3. Normal physical examination and a respiratory rate of 65/min:
• This suggests mild respiratory distress. TTN is characterized by mild respiratory distress in the first few hours of life, which typically resolves within 24–48 hours after birth.

Why Not the Other Options?
• a. Congenital pneumonia:
• This would typically present with more severe symptoms, including fever, poor feeding, and respiratory distress. The baby is vigorous, and the symptoms are not severe enough to suggest pneumonia.
• b. Pulmonary hypoplasia:
• Pulmonary hypoplasia is often associated with severe respiratory distress, abnormal chest findings, and typically occurs in the setting of congenital anomalies (e.g., diaphragmatic hernia or oligohydramnios). This baby’s normal examination and Apgar scores make this less likely.
• d. Meconium aspiration syndrome:
• Meconium aspiration syndrome typically presents with more significant respiratory distress, cyanosis, and grunting shortly after birth. It often presents with abnormal chest findings and requires more intensive respiratory support. This baby does not have those features.

Key Points:
• Transient Tachypnea of the Newborn (TTN) is a common cause of mild, self-limited tachypnea in the first hours of life. It occurs more commonly in cesarean deliveries and resolves within 24–48 hours.
• Management is typically supportive with oxygen and monitoring, and most infants improve without specific treatment.

How well did you know this?
1
Not at all
2
3
4
5
Perfectly
23
Q

A two-year-old male toddler was presented to an
EOPD with abnormal body movement which lasts
ten minutes for the first time. On evaluation, the
child started to convulse again. His RBS was 112
mg/dl. What is the most appropriate initial
management?
• a. Phenobarbital 20 mg/kg PO
b. Phosphenytoin 20 mg/kg V
c. Diazepam rectally 0.1ml/kg
• d. Dextrose 10percent 5ml/kg bolus

A

The correct answer is: c. Diazepam rectally 0.1 ml/kg.

Explanation:

This child is experiencing a convulsive episode (likely a seizure), and prompt management is required. Since the child is actively convulsing and the episode is ongoing, immediate treatment to stop the seizure is crucial.

Initial Management of Seizures:
• Diazepam is commonly used as an initial anticonvulsant for children with acute seizures, particularly when seizures are ongoing or recurrent. Diazepam is fast-acting and can be administered rectally in a child who is actively seizing and unable to take oral medication.

Why Not the Other Options?
• a. Phenobarbital 20 mg/kg PO:
• Phenobarbital is not used as first-line therapy for acute seizures in a child. It is a longer-acting anticonvulsant and would not be effective enough to control a seizure that is actively occurring.
• b. Phosphenytoin 20 mg/kg IV:
• Phenytoin (or its prodrug, phosphenytoin) is used for status epilepticus or when seizures are refractory to initial treatments, not for the initial management of a single seizure.
• d. Dextrose 10% 5ml/kg bolus:
• A low blood sugar (hypoglycemia) is not indicated in this case (RBS is 112 mg/dl, which is normal). Giving glucose in this situation would be unnecessary and not the most appropriate initial intervention.

Key Points:
• Diazepam is typically the first-line treatment for acute seizures to stop the convulsions.
• Rectal administration of diazepam is often used in children when intravenous access is not available or when seizures are ongoing.

24
Q

A two-year-old male toddler was presented to an
EOPD with abnormal body movement which lasts
ten minutes for the first time. On evaluation, the
child started to convulse again. His RBS was 112
mg/dl. What is the most appropriate initial
management?
• a. Phenobarbital 20 mg/kg PO
b. Phosphenytoin 20 mg/kg V
c. Diazepam rectally 0.1ml/kg
• d. Dextrose 10percent 5ml/kg bolus

A

The correct answer is: c. Diazepam rectally 0.1 ml/kg.

Explanation:

This child is experiencing a convulsive episode (likely a seizure), and prompt management is required. Since the child is actively convulsing and the episode is ongoing, immediate treatment to stop the seizure is crucial.

Initial Management of Seizures:
• Diazepam is commonly used as an initial anticonvulsant for children with acute seizures, particularly when seizures are ongoing or recurrent. Diazepam is fast-acting and can be administered rectally in a child who is actively seizing and unable to take oral medication.

Why Not the Other Options?
• a. Phenobarbital 20 mg/kg PO:
• Phenobarbital is not used as first-line therapy for acute seizures in a child. It is a longer-acting anticonvulsant and would not be effective enough to control a seizure that is actively occurring.
• b. Phosphenytoin 20 mg/kg IV:
• Phenytoin (or its prodrug, phosphenytoin) is used for status epilepticus or when seizures are refractory to initial treatments, not for the initial management of a single seizure.
• d. Dextrose 10% 5ml/kg bolus:
• A low blood sugar (hypoglycemia) is not indicated in this case (RBS is 112 mg/dl, which is normal). Giving glucose in this situation would be unnecessary and not the most appropriate initial intervention.

Key Points:
• Diazepam is typically the first-line treatment for acute seizures to stop the convulsions.
• Rectal administration of diazepam is often used in children when intravenous access is not available or when seizures are ongoing.

25
Q

An eight year old male child presents with a 10% weight loss over the previous 2 weeks and night time enuresis. On physical examination, he is sleepy and has rapid deep respiration. Investigations showed that blood glucose of 500mg/dl and urine ketone of 2+. What should be the first step in the management of this child?
Select one:
a. Regular insulin
b. Potassium chloride
c. Isotonic fluids
d. Sodium chloride

A

The correct answer is:

c. Isotonic fluids

Explanation:

This child presents with diabetic ketoacidosis (DKA), as evidenced by:
• Hyperglycemia (blood glucose of 500 mg/dL)
• Ketosis (urine ketones 2+)
• Weight loss, lethargy, and Kussmaul breathing (rapid, deep respirations indicating metabolic acidosis)

The first priority in the management of DKA is fluid resuscitation to correct dehydration, restore perfusion, and improve circulatory stability.

Steps in Management:
1. Isotonic fluids (e.g., 0.9% normal saline):
• Initiate immediately to restore intravascular volume.
• Avoid rapid correction of hyperglycemia to prevent cerebral edema.
• Typically, give 10-20 mL/kg bolus over the first hour.
2. Insulin therapy:
• Start regular insulin infusion (0.1 units/kg/hour) after initiating fluids.
• Do not administer insulin before fluids, as this may worsen dehydration and electrolyte imbalances.
3. Potassium replacement:
• Once serum potassium is known, begin potassium replacement if necessary. Potassium levels may appear normal or elevated initially, but total body potassium is depleted.
4. Sodium chloride:
• Use it as part of isotonic fluid replacement.

Summary:

Isotonic fluids (option c) are the first and most critical step in managing DKA in this child.

26
Q

Eight months old HIV exposed infant is on follow up at ART clinic. His DNA PCR done at 6 weeks of age was negative. The nurse advised the mother to continue breast feeding and co-trimoxazole prophylaxis. What should be your response if the mother asks you when to stop the co-trimoxazole?
Select one:
a. Continue until HAART is started
b. Continue until the age of 18 months
c. Continue until HIV infection is excluded
d. Continue until the age of 5 years

A

Answer is C
HIV-exposed infants who continue to breast feed should be provided with cotrimoxazole prophylaxis and should be re tested a minimum of six weeks after complete cessation of breastfeeding before HIV infection can be excluded. In addition, the infant should be re tested at any stage during breastfeeding if features of HIV infection occur.

The correct answer is:
c. Continue until HIV infection is excluded

Explanation:

Co-trimoxazole prophylaxis is given to HIV-exposed infants to prevent opportunistic infections, particularly Pneumocystis jirovecii pneumonia (PJP), which is a common and potentially fatal infection in immunocompromised children.
1. HIV-exposed infants are those born to HIV-positive mothers but whose HIV status is not yet confirmed. A negative DNA PCR at 6 weeks is reassuring but not definitive, as it does not rule out late transmission (e.g., through breastfeeding).
2. Co-trimoxazole prophylaxis should be continued until HIV infection is definitively excluded, which typically requires:
• A negative HIV antibody test at 18 months of age (or after breastfeeding cessation, whichever is later).
3. If HIV infection is confirmed at any point, the child will need to start HAART (antiretroviral therapy), and co-trimoxazole prophylaxis may be continued based on immune status (e.g., low CD4 count).

Key Points:
• Co-trimoxazole should not be stopped based on the negative DNA PCR alone at 6 weeks because there is still a risk of postnatal transmission via breastfeeding.
• Monitoring and follow-up are critical to determine the infant’s HIV status before discontinuing prophylaxis.

27
Q

Full-term infant is born after a normal pregnancy and delivery; however, it is complicated by marginal placental separation. At 12 hours of age, the infant, although appearing to be in good health, has vomiting of a coffee ground matter. What is the most likely diagnostic procedures that should be performed first to determine the cause of the bleeding?
Select one:
a. An Apt test
b. A barium enema
c. A platelet count, prothrombin time, and partial thromboplastin time.
d. Gastric lavage with normal saline

A

The correct answer is:
a. An Apt test

Explanation:

This infant is vomiting coffee ground material, which suggests the presence of blood in the stomach. The most likely cause of this presentation in a newborn is swallowed maternal blood, either during delivery or from the mother’s nipples during breastfeeding.

Diagnostic Approach:
1. Apt test:
• The Apt test distinguishes fetal hemoglobin (HbF) from adult hemoglobin (HbA) in the blood.
• If the blood is from the infant (fetal origin), further investigation is needed to identify the cause of bleeding.
• If the blood is from the mother (adult origin), no further workup is required for the infant.
2. Other options:
• Barium enema: Not appropriate in this scenario as there are no signs of lower gastrointestinal issues or obstruction.
• Platelet count, PT, PTT: Useful for evaluating coagulopathy, but not the first step since the bleeding is likely due to swallowed maternal blood.
• Gastric lavage with normal saline: This may clear the stomach contents, but it does not identify the source of the bleeding.

Summary:

The Apt test should be performed first to confirm whether the blood is of maternal or fetal origin, guiding the need for further evaluation.

28
Q

A mother has breast fed her newborn for 4 days. The newborn’s birth weight was 2100 g but he now weighs only 1900g. When she squeezes her nipple before feeding her baby, her milk appears very watery. What will be your advice to this mother?
Select one:
a. Start him on formula milk in addition to her breast milk
b. Start on exclusive formula feeding for the next 6 months
c. Continue breast feeding exclusively for the next 6 months
d. Start him on full strength cow’s milk in addition to her breast milk

A

The correct answer is:
c. Continue breastfeeding exclusively for the next 6 months

Explanation:

Breastfeeding is the ideal source of nutrition for a newborn, including low birth weight or small-for-gestational-age infants, unless there is a medical contraindication. Here’s why exclusive breastfeeding is recommended in this case:
1. Weight loss in the first few days:
• It is normal for newborns to lose up to 7-10% of their birth weight within the first few days due to fluid shifts and meconium passage.
• The weight loss here (9.5%) is within the acceptable range, but it requires close monitoring.
2. Breast milk composition:
• The milk appearing “watery” is normal; the first milk released during feeding is foremilk, which is high in water and lactose. The later milk (hindmilk) is richer in fat and calories.
• Encourage the mother to allow her baby to empty one breast fully before switching to the other, ensuring the infant receives hindmilk.
3. Support for breastfeeding:
• Educate the mother about proper latching and feeding techniques.
• Ensure the baby is fed frequently (8-12 times in 24 hours) to stimulate milk production and provide adequate calories.
4. Avoid formula or cow’s milk:
• Formula milk is unnecessary and may interfere with breastfeeding.
• Cow’s milk is not appropriate for infants under 1 year of age due to its high protein content, low iron, and risk of intestinal irritation.

Follow-up:

Monitor the baby’s weight gain and hydration status over the next few days. Most breastfed infants regain their birth weight by 10-14 days of age. If the baby continues to lose weight or shows signs of dehydration, further evaluation and support are needed.

29
Q

A neonate is delivered by difficult vacuum extraction. Ten hours after delivery, the neonate appears pale, has a heart rate of 180 beats per minute and is noticed to have a boggy swelling of the whole scalp. The packed cell volume and hemoglobin concentration are low. What is the correct treatment of this problem after admitting to NICU?
Select one:
a. Partial exchange transfusion
b. Maintenance fluid
c. IV antibiotics
d. Transfussion with whole blood

A

The correct answer is:
d. Transfusion with whole blood

Explanation:

This neonate likely has a subgaleal hemorrhage, a serious complication of vacuum-assisted delivery. It is characterized by bleeding into the subgaleal space, leading to significant blood loss and hypovolemic shock.

Key Features of Subgaleal Hemorrhage:
1. Boggy swelling: The swelling crosses suture lines and may extend over the entire scalp.
2. Pallor and tachycardia: Signs of hypovolemic shock due to blood loss.
3. Low hemoglobin/packed cell volume (PCV): Indicates significant anemia.
4. Risk of progression: Subgaleal hemorrhage can rapidly worsen and result in life-threatening hypovolemia.

Treatment:
1. Immediate whole blood transfusion:
• Corrects anemia and restores circulating blood volume.
• Whole blood is preferred over packed red blood cells (RBCs) in this case because it provides both red cells and plasma, addressing both oxygen-carrying capacity and volume deficit.
2. Supportive care in NICU:
• Monitor vital signs closely (heart rate, blood pressure, capillary refill).
• Ensure adequate oxygenation.
• Maintain fluid balance with IV fluids as necessary.
3. Additional interventions:
• Monitor coagulation parameters: Disseminated intravascular coagulation (DIC) can occur with severe hemorrhage.
• Administer fresh frozen plasma or other blood products if there are coagulation abnormalities.
• Consider antibiotics only if there is evidence of infection.

Why Not Other Options?
• a. Partial exchange transfusion: Used for polycythemia, not for acute anemia due to blood loss.
• b. Maintenance fluid: Insufficient to manage significant hypovolemia.
• c. IV antibiotics: Not indicated unless there is evidence of sepsis.

Summary:

The priority is to address the significant blood loss with whole blood transfusion, stabilizing the infant’s hemodynamic status and preventing further complications.

30
Q

A newly delivered female neonate developed failure to pass meconium of 48 hours duration with abdominal distension. On physical examination, she has pulse rate-120/min, she had hugely distended abdomen and has no anal opening. It was also seen that she had a single opening anteriorly with passage of feces but has no different urethral or vaginal opening. What would be the diagnosis?
Select one:
a. Imperforate anus with cloaca
b. Imperforate anus with rectovestibular fistula
c. Imperforate anus with perineal fistula
d. Imperforate anus with rectovaginal fistula

A

The correct answer is:
a. Imperforate anus with cloaca

Explanation:

This neonate presents with:
1. Imperforate anus: No visible anal opening.
2. Single opening anteriorly: This is a hallmark of a cloaca, where the rectum, vagina, and urethra form a single common channel.
3. Passage of feces through the single opening: Indicates the presence of a communication between the rectum and the urogenital system.

Key Features of the Diagnoses:
1. Imperforate anus with cloaca:
• In cloacal malformations, the rectum, vagina, and urethra converge into a single channel, forming a single opening.
• This is more common in females and is associated with abdominal distension and failure to pass meconium.
2. Imperforate anus with rectovestibular fistula:
• There would be a separate vaginal opening and a fistula connecting the rectum to the vestibule (area between the labia minora).
• In this case, there is no mention of a separate vaginal opening, ruling this out.
3. Imperforate anus with perineal fistula:
• Feces would pass through a fistula located on the perineum, near the normal location of the anus.
• This does not involve a single common opening.
4. Imperforate anus with rectovaginal fistula:
• There would be a separate urethral opening and a fistula connecting the rectum to the vagina.
• The absence of distinct vaginal and urethral openings makes this unlikely.

Diagnostic Confirmation:
• A pelvic ultrasound or MRI can confirm the anatomy of the malformation.
• A contrast study may also be used to delineate the anatomy of the cloacal malformation.

Management:
• Surgical correction is required, typically involving staged procedures:
1. Creation of a diverting colostomy to relieve obstruction.
2. Definitive repair to separate the rectum, vagina, and urethra.
• Multidisciplinary care involving pediatric surgeons, urologists, and gynecologists is essential.

31
Q

A nine month old infant presents with a 2 week history of paroxysmal cough, low-grade fever, post-tussive vomiting and nasal discharge. Physical examination reveals bilateral otitis media and conjunctival hemorrhages. Scattered inspiratory rales are present bilaterally. The complete blood count shows a WBC count of 45,000cells/mm3, 95% of which represent lymphocytes. What preventive measure would you take?
Select one:
a. DPT vaccination
b. Gamma globulin for high risk babies
c. Isolate the infant
d. Prophylactic antibiotic

A

C
But chatgpt says this

The presentation of paroxysmal cough, post-tussive vomiting, and a marked lymphocytosis (WBC count of 45,000 cells/mm³, with 95% lymphocytes) is highly suggestive of pertussis (whooping cough). Conjunctival hemorrhages and otitis media are common complications of severe coughing spells in pertussis.

Correct Answer: a. DPT vaccination

Rationale:
• Pertussis is a vaccine-preventable disease. The DPT vaccine (Diphtheria, Pertussis, Tetanus) is a key preventive measure to protect against pertussis in infancy.
• The infant likely has not been adequately vaccinated, as the complete DPT series is typically administered in multiple doses starting at 2 months of age, with subsequent doses at 4 and 6 months. Ensuring vaccination prevents future episodes of pertussis in the infant and others.

Why not the other options?
• b. Gamma globulin for high-risk babies: This is not indicated for pertussis prevention. Gamma globulin is typically used for passive immunity against specific infections (e.g., hepatitis or measles).
• c. Isolate the infant: While isolation may prevent transmission, it is not the primary preventive strategy. Treatment and vaccination are more effective long-term measures.
• d. Prophylactic antibiotic: Antibiotics like azithromycin are used to treat pertussis or as post-exposure prophylaxis for close contacts, but they do not replace vaccination as a preventive measure.

Vaccination remains the cornerstone of prevention for pertussis.

32
Q

A seven year old female HIV infected patient for the past 4 years was on follow up and taking her ART with poor adherence. She now presented with insidious onset of fast breathing and cough. She has generalized lymphadenopathy, wheezing, digital clubbing and hepato-splenomegaly. Her oxygen saturation is 85%. Chest X-ray showed bilateral reticulonodular infiltrates. What is the most likely diagnosis of her problem?
Select one:
a. Severe community acquired pneumonia
b. Milliary Tuberculosis
c. Lymphoid Interstitial Pneumonitis (LIP)
d. Pneumocystic Jeroveci Pneumonia (PJP)

A

The most likely diagnosis is:
c. Lymphoid Interstitial Pneumonitis (LIP)

Rationale:
• Lymphoid Interstitial Pneumonitis (LIP) is common in HIV-infected children, particularly in those with poor adherence to antiretroviral therapy.
• Key features in this case:
• HIV-infected child with poor ART adherence.
• Insidious onset of symptoms (suggesting a chronic process rather than acute pneumonia).
• Generalized lymphadenopathy, digital clubbing, and hepatosplenomegaly (classic findings in LIP).
• Oxygen saturation of 85% and bilateral reticulonodular infiltrates on chest X-ray are consistent with chronic interstitial lung disease.

Why not the other options?
• a. Severe community-acquired pneumonia:
• Pneumonia typically has an acute onset with fever and focal findings on chest X-ray (e.g., lobar consolidation). The insidious onset and diffuse infiltrates make this less likely.
• b. Miliary Tuberculosis:
• While TB is a consideration in HIV-infected children, miliary TB usually presents with systemic symptoms such as fever, weight loss, and diffuse “miliary” nodules on imaging. The presence of digital clubbing and chronic symptoms points more to LIP.
• d. Pneumocystis jirovecii Pneumonia (PJP):
• PJP presents acutely in younger infants (<6 months of age) or older children with severe immunosuppression. It is typically not associated with lymphadenopathy, clubbing, or hepatosplenomegaly, and imaging would show diffuse ground-glass opacities rather than reticulonodular infiltrates.

Summary:

LIP is a chronic interstitial lung disease seen in HIV-infected children, often due to chronic immune activation and poor viral suppression. It is the most likely diagnosis given the chronic nature of symptoms, systemic findings, and imaging results.

33
Q

A six month old female child was admitted and treated for acute meningitis. She improved initially. A few days after the onset of antibiotic therapy, she developed recurrence of high grade fever and projectile vomiting. Physical examination revealed increased head circumference, bulging fontanel, and separation of sutures, 2 episodes of abnormal body movement and change in mentation. What is most important lifesaving management of this child?
Select one:
a. Changing the antibiotics to more potent ones
b. Admit her to pediatric ICU for possible respiratory support
c. Neuro surgical consultation for their surgical intervention
d. Performing Subdural Tap

A

The most important lifesaving management for this child is:
d. Performing Subdural Tap

Rationale:

This child has clinical features of post-meningitic subdural effusion, a known complication of meningitis in infants. The key findings pointing to this diagnosis include:
• Recurrence of high-grade fever and projectile vomiting after initial improvement.
• Increased head circumference, bulging fontanel, and separation of sutures, suggesting increased intracranial pressure (ICP).
• Seizures and altered mentation, which indicate worsening neurological status due to the increased pressure.

A subdural tap is an immediate and critical intervention to relieve increased ICP caused by the collection of subdural fluid. This procedure is lifesaving in this situation.

Why not the other options?
• a. Changing the antibiotics to more potent ones:
• While appropriate antibiotics are critical, the acute problem here is the raised ICP. Delaying the reduction of ICP could lead to brain herniation and death, regardless of the antibiotic therapy.
• b. Admit her to pediatric ICU for possible respiratory support:
• ICU admission and respiratory support are important but do not directly address the life-threatening raised ICP caused by the subdural effusion.
• c. Neurosurgical consultation for surgical intervention:
• Neurosurgical consultation is important, but a subdural tap is an urgent bedside procedure that can provide immediate relief. Surgery is typically reserved for refractory cases or thick, loculated effusions.

Summary:

In this scenario, the most critical step is performing a subdural tap to reduce ICP and prevent further neurological damage or death.

34
Q

A six year old male child presented with fever and cough of 3 weeks duration. In addition, he developed vomiting, speech impairment and seizure in the last one week. On physical examination, he has nuchal rigidity, positive Kerning & Brudzinski signs, hypertonia and disorientation. CSF analysis showed WBC of 280 cells/mm3, L-82% & N-18%, glucose 40 mg/dl and protein is 95mg/dl. What is the most likely diagnosis?
Select one:
a. Tuberculous meningitis
b. Cryptococcal meningitis
c. Viral meningoencephalitis
d. Complicated bacterial meningitis

A

The most likely diagnosis is:
a. Tuberculous meningitis

Rationale:

This child presents with chronic symptoms (fever, cough for 3 weeks) and progressive neurological deterioration (vomiting, speech impairment, seizures, disorientation) along with classical signs of meningeal irritation (nuchal rigidity, positive Kernig’s and Brudzinski’s signs). The CSF analysis is highly suggestive of tuberculous meningitis (TBM):
• CSF findings:
• Elevated WBC count (280 cells/mm³) with a lymphocytic predominance (82% lymphocytes) — typical for TBM.
• Elevated protein (95 mg/dL) — indicates disruption of the blood-brain barrier.
• Low glucose (40 mg/dL) — consistent with TBM due to the high metabolic activity of Mycobacterium tuberculosis in the CNS.

Why not the other options?
• b. Cryptococcal meningitis:
• Cryptococcal meningitis is rare in immunocompetent children and more commonly occurs in HIV-infected or immunosuppressed individuals. CSF in cryptococcal meningitis usually shows lymphocytic predominance but often has very high opening pressure and positive India ink staining or Cryptococcal antigen test.
• c. Viral meningoencephalitis:
• Viral infections usually have normal or slightly elevated CSF protein, normal glucose, and mild lymphocytic pleocytosis. The chronicity of symptoms and significant protein elevation make viral etiology unlikely.
• d. Complicated bacterial meningitis:
• Bacterial meningitis typically shows a neutrophilic predominance, markedly elevated WBC count, very low glucose, and much higher protein levels. The chronic nature and lymphocytic predominance are not consistent with bacterial meningitis.

Summary:

The chronic nature of symptoms, CSF findings (lymphocytic pleocytosis, elevated protein, and low glucose), and neurological signs strongly point to tuberculous meningitis, which requires urgent treatment with anti-tuberculous therapy and corticosteroids to reduce inflammation and prevent complications.

35
Q

A term female newborn weighing 2400 gram is delivered at Ghandi Hospital, by spontaneous vaginal delivery with Apgar scores of 2 and 3 in the 1st and 5th minutes, respectively. Respiratory distress was apparent in the delivery room, with diminished breath sounds on the left side of chest and heart sounds are displaced to the right and the abdomen looks scaphoid. What is the underlying cause of the problem of this Newborn?
Select one:
a. Intra uterine growth restriction
b. Respiratory Distress Syndrome
c. Congenital Heart Disease
d. Diaphragmatic Hernia

A

The most likely underlying cause is:
d. Diaphragmatic Hernia

Rationale:

This newborn presents with classic findings suggestive of a congenital diaphragmatic hernia (CDH):
• Respiratory distress at birth.
• Diminished breath sounds on the left side and displaced heart sounds to the right, indicating mediastinal shift.
• Scaphoid abdomen, which occurs because abdominal contents (e.g., intestines, stomach) herniate into the chest cavity through a diaphragmatic defect, leaving the abdominal cavity underfilled.

CDH is a life-threatening condition that requires immediate stabilization and surgical correction. The defect most commonly occurs on the left side, leading to pulmonary hypoplasia and respiratory distress.

Why not the other options?
• a. Intrauterine growth restriction (IUGR):
• While the baby’s weight of 2400 g is slightly low for term, IUGR does not explain the specific findings of respiratory distress, mediastinal shift, or scaphoid abdomen.
• b. Respiratory Distress Syndrome (RDS):
• RDS is due to surfactant deficiency and typically occurs in preterm infants. It causes respiratory distress but does not explain the physical findings like a scaphoid abdomen or displaced heart sounds.
• c. Congenital Heart Disease (CHD):
• CHD may cause cyanosis and respiratory distress but would not present with diminished breath sounds, mediastinal shift, or scaphoid abdomen.

Summary:

The combination of respiratory distress, displaced heart sounds, diminished breath sounds on one side, and scaphoid abdomen strongly indicates congenital diaphragmatic hernia

36
Q

A three year old male, unvaccinated child awakens at night with a fever of 39.5oC and cough. On physical examination, he has drooling of saliva and stridor. What is the optimal management of this child?
Select one:
a. Immediate admission for possible intubation and iv antibiotics
b. Immediate admission and administration of aminophylline
c. Immediate admission and inhalation therapy with racemic epinephrine
d. Treatment with oral antibiotics

A

The correct answer is:
a. Immediate admission for possible intubation and IV antibiotics

Rationale:

This child presents with fever, stridor, drooling of saliva, and distress, which strongly suggests acute epiglottitis, a life-threatening condition that requires immediate airway management. Key points include:
• Unvaccinated status: Makes Haemophilus influenzae type b (Hib), a common cause of epiglottitis, more likely.
• Drooling and stridor: Classic signs of severe upper airway obstruction.
• High fever and sudden onset: Consistent with the rapid progression of epiglottitis.

The priority in managing epiglottitis is ensuring the airway remains open. This may require intubation or, in extreme cases, an emergency tracheostomy. IV antibiotics (e.g., ceftriaxone or cefotaxime) are necessary to treat the infection after the airway is secured.

Why not the other options?
• b. Immediate admission and administration of aminophylline:
• Aminophylline is used for lower airway obstruction (e.g., asthma) and has no role in the management of epiglottitis.
• c. Immediate admission and inhalation therapy with racemic epinephrine:
• Racemic epinephrine is effective for croup (laryngotracheobronchitis), but epiglottitis is caused by supraglottic inflammation, not subglottic edema. Racemic epinephrine will not relieve the obstruction caused by a swollen epiglottis.
• d. Treatment with oral antibiotics:
• Oral antibiotics are insufficient for a condition as severe and rapidly progressive as epiglottitis. Immediate IV antibiotics are required after securing the airway.

Summary:

This child likely has acute epiglottitis, which is a medical emergency. The first step is securing the airway through possible intubation and administering IV antibiotics to treat the infection. Avoid unnecessary agitation or procedures that might worsen airway obstruction.

37
Q

A three years old male child is brought to your pediatric OPD with an occasional abdominal pain and distention of a week duration. On Physical examination, the vital signs are all normal except slight abdominal distention. Stool examination revealed ova of Hymenolepis nana. What will be your management of this child?
Select one:
a. Niclosamide tabs once daily for 5 days
b. Stat dose of Mebendazole
c. Stat dose of Niclosamide tabs
d. Albendazol syrup for 3 days

A

The correct answer is:
d. Albendazole syrup for 3 days

Rationale:

Hymenolepis nana (dwarf tapeworm) is a common intestinal parasite in children. It is transmitted through contaminated food and water and can cause abdominal pain, distension, diarrhea, or be asymptomatic. Albendazole is the treatment of choice because it is highly effective against Hymenolepis nana. The typical dosage is 400 mg daily for 3 days, adjusted for weight and age in pediatric patients, and given as syrup for young children.

Why not the other options?
• a. Niclosamide tabs once daily for 5 days:
• Niclosamide is an older treatment for tapeworm infections, but it is less commonly used now because albendazole is more effective, easier to administer, and better tolerated.
• b. Stat dose of Mebendazole:
• Mebendazole is effective for other helminths like roundworms, hookworms, and whipworms but has limited efficacy against Hymenolepis nana and is not the drug of choice.
• c. Stat dose of Niclosamide tabs:
• A single dose of niclosamide is not sufficient for Hymenolepis nana infection, as it may not eradicate all stages of the parasite.

Summary:

The best treatment for Hymenolepis nana infection in a 3-year-old child is albendazole syrup for 3 days due to its efficacy and ease of use. Proper hygiene and prevention of reinfection are also critical to management.

38
Q

A three years old male child is brought to your pediatric OPD with an occasional abdominal pain and distention of a week duration. On Physical examination, the vital signs are all normal except slight abdominal distention. Stool examination revealed ova of Hymenolepis nana. What will be your management of this child?
Select one:
a. Niclosamide tabs once daily for 5 days
b. Stat dose of Mebendazole
c. Stat dose of Niclosamide tabs
d. Albendazol syrup for 3 days

A

The correct answer is:
d. Albendazole syrup for 3 days

Rationale:

Hymenolepis nana (dwarf tapeworm) is a common intestinal parasite in children. It is transmitted through contaminated food and water and can cause abdominal pain, distension, diarrhea, or be asymptomatic. Albendazole is the treatment of choice because it is highly effective against Hymenolepis nana. The typical dosage is 400 mg daily for 3 days, adjusted for weight and age in pediatric patients, and given as syrup for young children.

Why not the other options?
• a. Niclosamide tabs once daily for 5 days:
• Niclosamide is an older treatment for tapeworm infections, but it is less commonly used now because albendazole is more effective, easier to administer, and better tolerated.
• b. Stat dose of Mebendazole:
• Mebendazole is effective for other helminths like roundworms, hookworms, and whipworms but has limited efficacy against Hymenolepis nana and is not the drug of choice.
• c. Stat dose of Niclosamide tabs:
• A single dose of niclosamide is not sufficient for Hymenolepis nana infection, as it may not eradicate all stages of the parasite.

Summary:

The best treatment for Hymenolepis nana infection in a 3-year-old child is albendazole syrup for 3 days due to its efficacy and ease of use. Proper hygiene and prevention of reinfection are also critical to management.

39
Q

A two month old male infant had non-billous vomiting of two weeks duration. On examination he has visible peristaltic wave in left upper part of the abdomen. The most likely electrolyte abnormality is?
Select one:
a. Aciduria
b. Hypernatremia
c. Hyperkalemia
d. Metabolic acidosis

A

The most likely electrolyte abnormality is:
a. Aciduria

Rationale:

This infant’s presentation of non-bilious vomiting with a visible peristaltic wave is highly suggestive of infantile hypertrophic pyloric stenosis (IHPS). In this condition, projectile vomiting leads to significant fluid and electrolyte losses. The typical abnormalities include:
1. Hypokalemic, hypochloremic metabolic alkalosis:
• Vomiting results in loss of gastric acid (HCl), leading to alkalosis.
• Sodium and chloride losses are prominent, and potassium is lost through secondary mechanisms (renal compensation for alkalosis).
2. Paradoxical aciduria:
• Despite metabolic alkalosis, the kidneys excrete hydrogen ions in the urine (aciduria) due to sodium retention in exchange for hydrogen ions in an attempt to preserve fluid volume.

Why not the other options?
• b. Hypernatremia:
• Sodium is lost in vomiting, and dehydration usually leads to hyponatremia or normal sodium levels, not hypernatremia.
• c. Hyperkalemia:
• Hypokalemia is the typical finding due to renal loss of potassium during compensation for alkalosis. Hyperkalemia is not associated with pyloric stenosis.
• d. Metabolic acidosis:
• Pyloric stenosis results in metabolic alkalosis, not acidosis, due to the loss of acidic gastric contents.

Summary:

The hallmark electrolyte abnormalities in pyloric stenosis are hypokalemic, hypochloremic metabolic alkalosis with paradoxical aciduria. The management includes correcting electrolyte imbalances and fluid deficits before surgical intervention (pyloromyotomy).

40
Q

A two year old male toddler is brought to Pediatric OPD with a complaint of high grade fever, cough and nasal discharge of 3 days duration. On physical examination, PR=128, RR=50 and To= 38.5oC. He has bilateral red eyes, erythematous maculopapular skin rash starting from the retroauricular area then progress to involve the trunk. What is the most likely diagnosis?
Select one:
a. Measles
b. Meningococcemia
c. Sever community acquired pneumonia
d. Mumps

A

The most likely diagnosis is:
a. Measles

Rationale:

This child’s presentation of high-grade fever, cough, nasal discharge, and bilateral red eyes (conjunctivitis) along with a erythematous maculopapular rash starting from the retroauricular area and progressing to the trunk is characteristic of measles.

Key features include:
• Fever, cough, and nasal discharge are common early symptoms of measles.
• The conjunctivitis is a hallmark sign of measles, often accompanied by koplik spots (which may be present in the mouth).
• The rash typically starts at the hairline or behind the ears (retroauricular), and then spreads downward to involve the trunk and extremities in a cephalocaudal fashion.

Measles is a highly contagious viral illness, and vaccination (MMR) is critical for prevention.

Why not the other options?
• b. Meningococcemia:
• Meningococcemia typically presents with sepsis and petechial or purpuric rash, not a maculopapular rash. It would also be associated with more severe signs of shock and sepsis, which are not described here.
• c. Severe community-acquired pneumonia:
• While pneumonia can cause fever, cough, and difficulty breathing, it would not cause conjunctivitis or a maculopapular rash. Pneumonia also usually presents with respiratory distress and abnormal lung sounds on physical examination.
• d. Mumps:
• Mumps primarily causes parotitis (swelling of the parotid glands) and fever, but does not typically cause a rash like the one described here. It can cause some upper respiratory symptoms but is not associated with the maculopapular rash seen in measles.

Summary:

The most likely diagnosis is measles, given the combination of fever, cough, conjunctivitis, and the characteristic maculopapular rash that starts at the retroauricular area. Prompt diagnosis and isolation are important due to the contagious nature of measles. Vaccination is key to preventing the disease.

41
Q

A year old male patient came with right side undescended testis since birth. Upon examination, there is mass felt in the right inguinal region with absent testis on the right side of the scrotum. He was sent for confirmation and ultrasound revealed inguinal testis. What is the importance of orchidopexy?
Select one:
a. Decrease risk of malignancy
b. Increases risk of trauma
c. Improves fertility
d. Increases risk of torsion

A

The correct answer is:
a. Decrease risk of malignancy

Rationale:

Orchidopexy is the surgical procedure used to bring an undescended testis into the scrotum and secure it in place. The importance of orchidopexy, especially for an inguinal testis (testis located in the inguinal canal), includes several key benefits:
• Decreases the risk of malignancy: Undescended testes (cryptorchidism), especially those that remain in the inguinal canal, have a significantly increased risk of testicular cancer. Orchidopexy reduces this risk by bringing the testis into the scrotum, where it is exposed to the normal temperature and environment.
• Improves fertility: Although the testis may have decreased fertility potential due to being undescended, orchidopexy improves the chances of fertility compared to leaving the testis in the inguinal canal, where the higher temperature can impair spermatogenesis.

Why not the other options?
• b. Increases risk of trauma: Orchidopexy does not increase the risk of trauma. In fact, bringing the testis into the scrotum provides better protection from external trauma.
• c. Improves fertility: While orchidopexy may improve fertility potential, its primary role is in reducing malignancy risk and improving the environment for spermatogenesis, rather than directly improving fertility.
• d. Increases risk of torsion: Undescended testes are at higher risk for testicular torsion, but orchidopexy actually decreases this risk by fixing the testis in place, preventing it from twisting within the inguinal canal.

Summary:

The primary importance of orchidopexy is to decrease the risk of malignancy associated with undescended testis, particularly by securing the testis in the scrotum and improving its conditions for spermatogenesis. It also reduces the risk of torsion and trauma.

42
Q

An 18 year old male patient presented with facial swelling in the morning which progresses to become generalized body swelling. Lab investigation showed Hg= 13g/dl, Urine dipstick +3 protein, Serum Creatinine = 1mg/dl. What next best investigation will you do?
Select one:
a. 24 hr Urinary protein
b. Urinary protein to creatinine ratio
c. Sulfosalicylic acid test
d. Urinary albumin to creatinine ratio

A

The correct answer is:
a. 24 hr Urinary protein

Rationale:

The patient presents with facial swelling (which can be a sign of nephrotic syndrome), generalized body swelling, proteinuria (3+ protein on urine dipstick), and a normal creatinine level (1 mg/dl), which suggests an issue related to glomerular filtration. The next best investigation to quantify the amount of proteinuria and assess the severity of kidney involvement is a 24-hour urinary protein collection.
• 24-hour urinary protein is a gold standard test to quantify proteinuria and confirm whether it is in the nephrotic range (≥3.5 grams of protein per day).

Why not the other options?
• b. Urinary protein to creatinine ratio:
• The protein-to-creatinine ratio can be helpful for estimating proteinuria, but it is less accurate than a 24-hour urine collection in quantifying the exact amount of protein loss, especially in the context of potential nephrotic syndrome.
• c. Sulfosalicylic acid test:
• This test is used to detect precipitation of proteins in the urine and may help confirm the presence of proteinuria, but it does not provide a quantitative measure. It would not be the next best step for evaluating the extent of protein loss.
• d. Urinary albumin to creatinine ratio:
• The albumin-to-creatinine ratio is useful for detecting microalbuminuria (early kidney damage), particularly in diabetic nephropathy, but it is less specific for detecting the higher levels of proteinuria found in nephrotic syndrome.

Summary:

Given the patient’s presentation of generalized edema and moderate proteinuria, a 24-hour urinary protein collection is the best next step to evaluate the extent of proteinuria and guide management, particularly to confirm a diagnosis like nephrotic syndrome.

43
Q

A 9-year-old boy is being treated for acute osteomyelitis of the distal tibia with appropriate IV antibiotic therapy. After three days of treatment, he fails to show any clinical improvement. The patient subsequently undergone formal open surgical debridement, without complications. What serial evaluation is the most reliable method to determine the early success of treatment?
Select one:
a. C-reactive protein
b. Erythrocyte sedimentation rate
c. White blood cell count
d. Radiographs

A

The most reliable serial evaluation to determine the early success of treatment for osteomyelitis is a. C-reactive protein (CRP).

CRP is an acute-phase reactant that increases in response to infection and inflammation. It tends to decrease more quickly than the erythrocyte sedimentation rate (ESR) and white blood cell count after appropriate treatment. CRP levels are therefore a useful marker for monitoring the response to therapy in cases of osteomyelitis. In contrast, ESR and WBC count may take longer to normalize and may not reflect clinical improvement as reliably. Radiographs are typically not useful for detecting early changes in osteomyelitis treatment response.

44
Q

A four year old girl has multiple joint swelling for over 6 months. She is slow to move in the morning and moves as if stiff for the first hours of the day. Then after, she will become very active. She has no rash and very little limitation of range of motion. Her ESR is 40 mm/hr. What is the most likely diagnosis?
Select one:
a. Juvenile rheumatoid arthritis
b. Systemic lupus erythematosus
c. Septic arthritis
d. Reactive arthritis

A

The most likely diagnosis for this 4-year-old girl is a. Juvenile rheumatoid arthritis (JRA), now commonly referred to as juvenile idiopathic arthritis (JIA).

The key features that suggest this diagnosis are:
• Multiple joint swelling for over 6 months: JIA is characterized by persistent joint inflammation lasting for more than 6 weeks, typically affecting multiple joints.
• Morning stiffness: This is a hallmark of inflammatory conditions like JIA.
• Active later in the day: Joint stiffness often improves as the day progresses in JIA, especially after some movement.
• Elevated ESR: An elevated ESR is a nonspecific marker of inflammation, which is often seen in JIA.

The absence of a rash, systemic symptoms, and a clear infectious origin (as seen in septic arthritis or reactive arthritis) further supports JIA as the most likely diagnosis.

Here’s why the other options are less likely in this case:

b. Systemic lupus erythematosus (SLE)
• SLE can cause joint swelling and morning stiffness, but it is typically accompanied by other systemic symptoms like a malar rash (butterfly rash), photosensitivity, oral ulcers, and signs of renal or neurological involvement. The absence of a rash and other systemic features makes SLE less likely in this case.

c. Septic arthritis
• Septic arthritis is usually an acute, severe condition that presents with severe joint pain, redness, warmth, and limited range of motion. It often involves a single joint (though it can involve multiple joints in some cases), and it is associated with systemic signs of infection, such as fever and leukocytosis (high white blood cell count). This child has no fever and only mild joint swelling, which makes septic arthritis unlikely.

d. Reactive arthritis
• Reactive arthritis typically follows an infection (e.g., gastrointestinal or genitourinary) and is usually acute in onset, often with a characteristic triad of symptoms: arthritis, urethritis, and conjunctivitis. This patient does not have any history of preceding infection, nor does she have the characteristic symptoms of reactive arthritis, making it less likely.

In conclusion, juvenile idiopathic arthritis (JIA) fits best with the chronic, non-acute joint swelling and stiffness, along with an elevated ESR and the absence of other systemic symptoms.

45
Q

A 6 month old male infant is presented to Pediatric OPD with cough and fast breathing of 2 weeks duration. His mother also explained that he had excessive sweating and interruption during breast feeding. On P/E, he is in respiratory distress with RR=68/min, PR= 144/Min and T = 38.50C. He has intercostal and subcostal retractions, Grade III Systolic murmur best heard at LUSB and tender hepatomegaly. What is the most likely Cardiac lesion of this infant?
Select one:
a. PDA
b. VSD
c. PS
d. ASD

A

The clinical presentation of this infant suggests congestive heart failure (CHF) caused by a congenital heart defect. Let’s analyze the key findings:
1. Symptoms:
• Cough and fast breathing for 2 weeks indicate respiratory distress, likely due to pulmonary congestion or infections secondary to heart failure.
• Excessive sweating and interruption during feeding are classic signs of CHF in infants.
2. Physical Examination:
• Tachypnea (RR = 68/min) and retractions indicate significant respiratory distress.
• Grade III systolic murmur best heard at the left upper sternal border (LUSB).
• Tender hepatomegaly, a sign of right-sided heart failure.
3. Most likely cardiac lesion:
• A patent ductus arteriosus (PDA) or a ventricular septal defect (VSD) would commonly cause these findings, as both lesions result in a left-to-right shunt, leading to pulmonary overcirculation and CHF.
• A systolic murmur at the LUSB strongly suggests PDA, as this is its characteristic auscultatory finding.
• VSD murmurs are typically best heard at the left lower sternal border (LLSB).

Answer: a. PDA

46
Q

A 5 year-old child fall from a tree and sustained injury to the right thigh. He has mild thigh swelling with intact neurovascular structure. Femoral x-ray showed a transverse mid shaft femoral fracture with a centimeter shorting. What is most acceptable way of treating his fracture?
Select one:
a. Immediate hip Spica casting
b. Flexible intramedullary nail
c. Skin traction for a week then hip spica casting
d. Open reduction and plating

A

The most acceptable treatment for a transverse mid-shaft femoral fracture in a 5-year-old child depends on the child’s age, fracture characteristics, and the presence of shortening. Here’s an analysis of the options:
1. Key findings:
• Transverse mid-shaft femoral fracture with mild swelling and 1 cm shortening.
• Neurovascular structures are intact, suggesting no immediate need for surgical exploration.
• The patient is 5 years old, an age where non-surgical options can often be considered.
2. Best treatment:
• For children under 6 years old with femoral fractures, non-surgical management is preferred due to the high remodeling potential of their bones.
• Skin traction followed by hip spica casting allows the initial reduction of the fracture (via traction) and stabilization for healing.
• Immediate hip spica casting is less commonly used for mid-shaft fractures with significant shortening, as initial traction ensures proper alignment.
• Flexible intramedullary nailing is typically reserved for older children (around 6–12 years) with unstable or displaced fractures.
• Open reduction and plating is more invasive and generally reserved for cases where non-surgical or minimally invasive methods are contraindicated.

Answer: c. Skin traction for a week then hip spica casting

47
Q

A 3-year-old presents with a 24-hours history of limping and progressive inability to bear weight. Parents recall no history of trauma, but note that he recently had an upper respiratory infection. The patient’s vital signs are stable except for low grade fever. Physical examination is limited because of pain. An ultrasound found hip effusion with laboratory values of WBC-15,000 (97% PMN), ESR-120, and CRP-5.0 mg/dl. What is the next best step in managing this patient?
Select one:
a. Emergent hip arthrotomy
b. Obtain an MRI
c. Admit for observation
d. Start the patient on IV antibiotics

A

This 3-year-old child presents with acute onset of limping, inability to bear weight, and significant systemic inflammatory markers, concerning for septic arthritis of the hip, a surgical emergency. Here’s the reasoning behind the best next step:

Key Findings:
1. History:
• Acute onset of limping and inability to bear weight without trauma.
• Recent upper respiratory infection raises concern for transient synovitis but does not rule out septic arthritis.
2. Examination and Labs:
• Hip effusion on ultrasound.
• Elevated inflammatory markers (WBC = 15,000 with 97% PMN, ESR = 120, CRP = 5.0 mg/dl).
• Fever (even low-grade) increases suspicion for infection.
3. Kocher Criteria for Septic Arthritis (≥2 is concerning):
• Fever >38.5°C.
• Non-weight bearing on affected side.
• ESR >40 mm/h.
• WBC >12,000.
This child meets 3/4 criteria, making septic arthritis highly likely.

Management:
• Septic arthritis is a surgical emergency. Delayed treatment risks joint destruction and complications such as avascular necrosis of the femoral head.
• Emergent hip arthrotomy (surgical drainage and irrigation) is the next step to confirm the diagnosis, relieve pressure, and prevent joint damage.
• MRI is helpful if the diagnosis is uncertain, but given the clinical and laboratory findings, immediate intervention is warranted.
• Starting IV antibiotics is critical but should occur after joint aspiration or arthrotomy, as cultures are essential for guiding therapy.

Answer: a. Emergent hip arthrotomy

48
Q

All are diagnostic criteria for febrile seizure except
A. Prior history of afebrile seizure
B. Absence of CNS infection and other metabolic imbalance
C. Age between 6 month and 5 years
D. Temperature greater than 380c

A

A
Seizures in children aged 6 months to 5 years that are associated with fever (≥ 38°C (100.4°F) in the absence of CNS infection, metabolic abnormalities, or a history of afebrile seizures. May be classified based on clinical features of febrile seizures into: [1]

Simple febrile seizures (∼ 70% of all febrile seizures)
Complex febrile seizures (∼ 25%)
Febrile status epilepticus (∼ 5%)
FEEDBACK

49
Q

Which one of the following is preventable cause of seizure
A. Developmental disorders
B. Perinatal asphyxia
C. Genetic disorders
D. Idiopathic

A

The correct answer is B. Perinatal asphyxia.

Explanation:
• Perinatal asphyxia refers to oxygen deprivation to the brain during birth, which can lead to brain injury and seizures. This is considered a preventable cause of seizures with proper prenatal care, timely intervention during labor, and adequate neonatal care.
• Developmental disorders, genetic disorders, and idiopathic seizures are not typically preventable causes. These are often inherent or arise from unknown factors beyond immediate control.

50
Q

The commonest type of neonatal seizure is
A. Tonic clonic seizure
B. Atonic seizure
C. Absence seizure
D. Subtle seizure

A

The correct answer is D. Subtle seizure.

Explanation:

Subtle seizures are the most common type of neonatal seizures, especially in preterm and full-term neonates. They may present with subtle signs like:
• Lip smacking
• Eye deviation or blinking
• Pedaling movements of the legs
• Apnea or irregular breathing
• Grimacing or chewing movements

Other types:
• Tonic-clonic seizures (A): Rare in neonates due to immature brain development.
• Atonic seizures (B): Rare in neonates and more common in older children or adults.
• Absence seizures (C): Typically occur in older children and are not common in the neonatal period.

51
Q

Not a physiologic jaundice
A. Visible jaundice appearing after 24 hrs
B. Total bilirubin rise by >5mg/dl
C. Jaundice which disappears within 1 week in term and 2 weeks in preterm
D. Total bilirubin rise by <5mg/dl

A

B

52
Q
  1. A 2 year old male child presented with cough, high grade fever and coryza associated with this he has skin rash that progressed from head to trunk and redness of the eye of 3 days duration.
    On P/E- V/S- PR-144 RR-40 T-37.8

Integ – multiple maculo papular rashes over the face and trunk. All of the following are false about the above problem, except?

A. The most likely diagnosis is chickenpox
B. Mostly caused by toxic bacteria
C. It is not contagious
D. Infection confers life-long immunity

A

The correct answer is D. Infection confers life-long immunity.

Explanation:

The clinical presentation is consistent with measles (rubeola), a viral infection characterized by:
• Cough, coryza, and conjunctivitis (the “3 Cs”), along with fever and maculopapular rash.
• The rash typically starts on the face and progresses downward to involve the trunk and limbs.
• Measles is highly contagious, caused by the measles virus (not bacteria), and infection typically confers life-long immunity.

Incorrect options:
• A. The most likely diagnosis is chickenpox: False, as chickenpox rash is vesicular, not maculopapular, and spreads differently.
• B. Mostly caused by toxic bacteria: False, as measles is caused by a virus (measles virus).
• C. It is not contagious: False, measles is highly contagious, with transmission via respiratory droplets.

53
Q

Which of the following is complication of nephrotic syndrome?
A. Thrombosis
B. Spontaneous bacterial peritonitis
C. sepsis
D. All

A

D ALL

54
Q

Twin babies delivered via SVD after 37 weeks of gestation are currently doing fine with the mother. In comparison
with the first twin, the second twin (twin B) could have increased risk of developing some complications. For what
complication is twin B more at risk?
at o f
N e s t i e s
© a. Hypoglycemia
• b. Neonatal sepsis
© c. Polycythemia
• d. Perinatal asphyxia

A

The correct answer is:

d. Perinatal asphyxia

Twin B is at a higher risk of developing perinatal asphyxia compared to Twin A. This increased risk arises from the potential complications associated with the delivery of the second twin, such as delayed delivery after Twin A, cord prolapse, uterine contractions causing reduced blood flow, or malpresentation. These factors can compromise oxygen delivery to Twin B, leading to asphyxia.

Other complications like hypoglycemia, neonatal sepsis, or polycythemia are not as directly linked to the birth order of twins.

55
Q

primiparous mother has brought her three -day-old male neonate to a neonatal OPD after she noticed yellowish
discoloration of his body since the day before. His birth weight was 3200 gm. On examination, he has tachycardia and
signs of dehydration, otherwise all neonatal reflexes are intact. His current weight is 250 gm, total serum bilirubin
measures 15 mg/di and indirect one is 14 mg/di. What is the most likely cause of jaundice in this case?
a. Neonatal sepsis
Cb. Breast feeding
C c Neonatal hepatitis
0 d . Breast milk

A

The most likely cause of jaundice in this case is:

b. Breastfeeding

Explanation:

The neonate has signs of breastfeeding jaundice, which typically occurs due to insufficient breastfeeding, leading to dehydration and decreased clearance of bilirubin. Key points in this case:
1. Weight loss: The neonate has lost more than 7% of his birth weight, indicating inadequate breastfeeding.
2. Dehydration: Signs of dehydration (tachycardia and decreased weight) suggest inadequate fluid intake.
3. Indirect hyperbilirubinemia: A total bilirubin of 15 mg/dL with a predominantly indirect fraction (14 mg/dL) is consistent with physiologic or breastfeeding jaundice.

Other options:
• a. Neonatal sepsis: Unlikely because the baby has no other signs of infection (e.g., lethargy, temperature instability, poor feeding, or irritability).
• c. Neonatal hepatitis: Rare in this age group and would typically present with direct hyperbilirubinemia.
• d. Breast milk jaundice: Usually develops later (after 4-7 days of life) and is not associated with dehydration or significant weight loss.

Management:
• Ensure adequate breastfeeding (8–12 times/day).
• Monitor weight, hydration, and bilirubin levels.
• Phototherapy may be needed if bilirubin levels approach risk thresholds for kernicterus.

56
Q

A five-year-old female child was brought to an emergency OPD after a high-speed vehicle hit her. On primary survey,
air way was patent and C-spine was stabilized, chest was clear with good air entry, but her blood pressure was un-
recordable and pulse was feeble. However, there was no visible external bleeding. The nurse faced difficulty to secure
IV line. What is the most appropriate and rapid IV site to resuscitate this patient?
a. Venous cut down
b. Intra-osseous
• c. Mid-cubital vein
d. Central line insertio

A

The most appropriate and rapid IV site to resuscitate this patient is:

b. Intra-osseous (IO)

Explanation:

In emergency situations where intravenous (IV) access is difficult or impossible, intra-osseous access is the fastest and most reliable method for delivering fluids and medications, especially in children.
1. Indications for IO access:
• Severe hypovolemia or shock (as seen in this patient with unrecordable blood pressure and feeble pulse).
• Failure to establish IV access within 60–90 seconds.
2. Common IO sites:
• Proximal tibia (most commonly used).
• Distal femur.
• Proximal humerus.
3. Advantages of IO access:
• Quick and easy to establish.
• Effective for fluid resuscitation, blood products, and medication administration.

Other options:
• a. Venous cut down: This is a surgical procedure requiring time and skill, making it less suitable in emergencies.
• c. Mid-cubital vein: Difficult to access in children with shock due to collapsed veins.
• d. Central line insertion: Time-consuming and technically challenging in critically ill children.

Thus, intra-osseous access is the preferred choice for rapid resuscitation in this scenario.